logo-type-white

AP® Biology

The chi square test: ap® biology crash course.

  • The Albert Team
  • Last Updated On: March 7, 2024

The Chi Square Test - AP® Biology Crash Course

The statistics section of the AP® Biology exam is without a doubt one of the most notoriously difficult sections. Biology students are comfortable with memorizing and understanding content, which is why this topic seems like the most difficult to master. In this article,  The Chi Square Test: AP® Biology Crash Course , we will teach you a system for how to perform the Chi Square test every time. We will begin by reviewing some topics that you must know about statistics before you can complete the Chi Square test. Next, we will simplify the equation by defining each of the Chi Square variables. We will then use a simple example as practice to make sure that we have learned every part of the equation. Finally, we will finish with reviewing a more difficult question that you could see on your AP® Biology exam .

Null and Alternative Hypotheses

As background information, first you need to understand that a scientist must create the null and alternative hypotheses prior to performing their experiment. If the dependent variable is not influenced by the independent variable , the null hypothesis will be accepted. If the dependent variable is influenced by the independent variable, the data should lead the scientist to reject the null hypothesis . The null and alternative hypotheses can be a difficult topic to describe. Let’s look at an example.

Consider an experiment about flipping a coin. The null hypothesis would be that you would observe the coin landing on heads fifty percent of the time and the coin landing on tails fifty percent of the time. The null hypothesis predicts that you will not see a change in your data due to the independent variable.

The alternative hypothesis for this experiment would be that you would not observe the coins landing on heads and tails an even number of times. You could choose to hypothesize you would see more heads, that you would see more tails, or that you would just see a different ratio than 1:1. Any of these hypotheses would be acceptable as alternative hypotheses.

Defining the Variables

Now we will go over the Chi-Square equation. One of the most difficult parts of learning statistics is the long and confusing equations. In order to master the Chi Square test, we will begin by defining the variables.

This is the Chi Square test equation. You must know how to use this equation for the AP® Bio exam. However, you will not need to memorize the equation; it will be provided to you on the AP® Biology Equations and Formulas sheet that you will receive at the beginning of your examination.

chi square

Now that you have seen the equation, let’s define each of the variables so that you can begin to understand it!

•   X 2  :The first variable, which looks like an x, is called chi squared. You can think of chi like x in algebra because it will be the variable that you will solve for during your statistical test. •   ∑ : This symbol is called sigma. Sigma is the symbol that is used to mean “sum” in statistics. In this case, this means that we will be adding everything that comes after the sigma together. •   O : This variable will be the observed data that you record during your experiment. This could be any quantitative data that is collected, such as: height, weight, number of times something occurs, etc. An example of this would be the recorded number of times that you get heads or tails in a coin-flipping experiment. •   E : This variable will be the expected data that you will determine before running your experiment. This will always be the data that you would expect to see if your independent variable does not impact your dependent variable. For example, in the case of coin flips, this would be 50 heads and 50 tails.

The equation should begin to make more sense now that the variables are defined.

Working out the Coin Flip

We have talked about the coin flip example and, now that we know the equation, we will solve the problem. Let’s pretend that we performed the coin flip experiment and got the following data:

Now we put these numbers into the equation:

Heads (55-50) 2 /50= .5

Tails (45-50) 2 /50= .5

Lastly, we add them together.

c 2 = .5+.5=1

Now that we have c 2 we must figure out what that means for our experiment! To do that, we must review one more concept.

Degrees of Freedom and Critical Values

Degrees of freedom is a term that statisticians use to determine what values a scientist must get for the data to be significantly different from the expected values. That may sound confusing, so let’s try and simplify it. In order for a scientist to say that the observed data is different from the expected data, there is a numerical threshold the scientist must reach, which is based on the number of outcomes and a chosen critical value.

Let’s return to our coin flipping example. When we are flipping the coin, there are two outcomes: heads and tails. To get degrees of freedom, we take the number of outcomes and subtract one; therefore, in this experiment, the degree of freedom is one. We then take that information and look at a table to determine our chi-square value:

how to write a null hypothesis ap bio

We will look at the column for one degree of freedom. Typically, scientists use a .05 critical value. A .05 critical value represents that there is a 95% chance that the difference between the data you expected to get and the data you observed is due to something other than chance. In this example, our value will be 3.84.

Coin Flip Results

In our coin flip experiment, Chi Square was 1. When we look at the table, we see that Chi Square must have been greater than 3.84 for us to say that the expected data was significantly different from the observed data. We did not reach that threshold. So, for this example, we will say that we failed to reject the null hypothesis.

The best way to get better at these statistical questions is to practice. Next, we will go through a question using the Chi Square Test that you could see on your AP® Bio exam.

AP® Biology Exam Question

This question was adapted from the 2013 AP® Biology exam.

In an investigation of fruit-fly behavior, a covered choice chamber is used to test whether the spatial distribution of flies is affected by the presence of a substance placed at one end of the chamber. To test the flies’ preference for glucose, 60 flies are introduced into the middle of the choice chamber at the insertion point. A ripe banana is placed at one end of the chamber, and an unripe banana is placed at the other end. The positions of flies are observed and recorded after 1 minute and after 10 minutes. Perform a Chi Square test on the data for the ten minute time point. Specify the null hypothesis and accept or reject it.

1211821
1045312

Okay, we will begin by identifying the null hypothesis . The null hypothesis would be that the flies would be evenly distributed across the three chambers (ripe, middle, and unripe).

Next, we will perform the Chi-Square test just like we did in the heads or tails experiment. Because there are three conditions, it may be helpful to use this set up to organize yourself:

  /E
  

Ok, so we have a Chi Square of 48.9. Our degrees of freedom are 3(ripe, middle, unripe)-1=2. Let’s look at that table above for a confidence variable of .05. You should get a value of 5.99. Our Chi Square value of 48.9 is much larger than 5.99 so in this case we are able to reject the null hypothesis. This means that the flies are not randomly assorting themselves, and the banana is influencing their behavior.

The Chi Square test is something that takes practice. Once you learn the system of solving these problems, you will be able to solve any Chi Square problem using the exact same method every time! In this article, we have reviewed the Chi Square test using two examples. If you are still interested in reviewing the bio-statistics that will be on your AP® Biology Exam, please check out our article The Dihybrid Cross Problem: AP® Biology Crash Course . Let us know how studying is going and if you have any questions!

Need help preparing for your AP® Biology exam?

AP® Biology practice questions

Albert has hundreds of AP® Biology practice questions, free response, and full-length practice tests to try out.

Interested in a school license?​

Popular posts.

AP® Physics I score calculator

AP® Score Calculators

Simulate how different MCQ and FRQ scores translate into AP® scores

how to write a null hypothesis ap bio

AP® Review Guides

The ultimate review guides for AP® subjects to help you plan and structure your prep.

how to write a null hypothesis ap bio

Core Subject Review Guides

Review the most important topics in Physics and Algebra 1 .

how to write a null hypothesis ap bio

SAT® Score Calculator

See how scores on each section impacts your overall SAT® score

how to write a null hypothesis ap bio

ACT® Score Calculator

See how scores on each section impacts your overall ACT® score

how to write a null hypothesis ap bio

Grammar Review Hub

Comprehensive review of grammar skills

how to write a null hypothesis ap bio

AP® Posters

Download updated posters summarizing the main topics and structure for each AP® exam.

Null hypothesis

null hypothesis definition

Null hypothesis n., plural: null hypotheses [nʌl haɪˈpɒθɪsɪs] Definition: a hypothesis that is valid or presumed true until invalidated by a statistical test

Table of Contents

Null Hypothesis Definition

Null hypothesis is defined as “the commonly accepted fact (such as the sky is blue) and researcher aim to reject or nullify this fact”.

More formally, we can define a null hypothesis as “a statistical theory suggesting that no statistical relationship exists between given observed variables” .

In biology , the null hypothesis is used to nullify or reject a common belief. The researcher carries out the research which is aimed at rejecting the commonly accepted belief.

What Is a Null Hypothesis?

A hypothesis is defined as a theory or an assumption that is based on inadequate evidence. It needs and requires more experiments and testing for confirmation. There are two possibilities that by doing more experiments and testing, a hypothesis can be false or true. It means it can either prove wrong or true (Blackwelder, 1982).

For example, Susie assumes that mineral water helps in the better growth and nourishment of plants over distilled water. To prove this hypothesis, she performs this experiment for almost a month. She watered some plants with mineral water and some with distilled water.

In a hypothesis when there are no statistically significant relationships among the two variables, the hypothesis is said to be a null hypothesis. The investigator is trying to disprove such a hypothesis. In the above example of plants, the null hypothesis is:

There are no statistical relationships among the forms of water that are given to plants for growth and nourishment.

Usually, an investigator tries to prove the null hypothesis wrong and tries to explain a relation and association between the two variables.

An opposite and reverse of the null hypothesis are known as the alternate hypothesis . In the example of plants the alternate hypothesis is:

There are statistical relationships among the forms of water that are given to plants for growth and nourishment.

The example below shows the difference between null vs alternative hypotheses:

Alternate Hypothesis: The world is round Null Hypothesis: The world is not round.

Copernicus and many other scientists try to prove the null hypothesis wrong and false. By their experiments and testing, they make people believe that alternate hypotheses are correct and true. If they do not prove the null hypothesis experimentally wrong then people will not believe them and never consider the alternative hypothesis true and correct.

The alternative and null hypothesis for Susie’s assumption is:

  • Null Hypothesis: If one plant is watered with distilled water and the other with mineral water, then there is no difference in the growth and nourishment of these two plants.
  • Alternative Hypothesis:  If one plant is watered with distilled water and the other with mineral water, then the plant with mineral water shows better growth and nourishment.

The null hypothesis suggests that there is no significant or statistical relationship. The relation can either be in a single set of variables or among two sets of variables.

Most people consider the null hypothesis true and correct. Scientists work and perform different experiments and do a variety of research so that they can prove the null hypothesis wrong or nullify it. For this purpose, they design an alternate hypothesis that they think is correct or true. The null hypothesis symbol is H 0 (it is read as H null or H zero ).

Why is it named the “Null”?

The name null is given to this hypothesis to clarify and explain that the scientists are working to prove it false i.e. to nullify the hypothesis. Sometimes it confuses the readers; they might misunderstand it and think that statement has nothing. It is blank but, actually, it is not. It is more appropriate and suitable to call it a nullifiable hypothesis instead of the null hypothesis.

Why do we need to assess it? Why not just verify an alternate one?

In science, the scientific method is used. It involves a series of different steps. Scientists perform these steps so that a hypothesis can be proved false or true. Scientists do this to confirm that there will be any limitation or inadequacy in the new hypothesis. Experiments are done by considering both alternative and null hypotheses, which makes the research safe. It gives a negative as well as a bad impact on research if a null hypothesis is not included or a part of the study. It seems like you are not taking your research seriously and not concerned about it and just want to impose your results as correct and true if the null hypothesis is not a part of the study.

Development of the Null

In statistics, firstly it is necessary to design alternate and null hypotheses from the given problem. Splitting the problem into small steps makes the pathway towards the solution easier and less challenging. how to write a null hypothesis?

Writing a null hypothesis consists of two steps:

  • Firstly, initiate by asking a question.
  • Secondly, restate the question in such a way that it seems there are no relationships among the variables.

In other words, assume in such a way that the treatment does not have any effect.

Questions Null Hypothesis
Are adults doing better at mathematics than teenagers? Mathematical ability does not depend on age.
Does the risk of a heart attack reduce by daily intake of aspirin? A heart attack is not affected by the daily dose of aspirin.
Are teenagers using cell phones to access the internet more than elders? Age does not affect the usage of cell phones for internet access.
Are cats concerned about their food color? Cats do not prefer food based on color.
Does pain relieve by chewing willow bark? Pain is not relieved by chewing willow bark.

The usual recovery duration after knee surgery is considered almost 8 weeks.

A researcher thinks that the recovery period may get elongated if patients go to a physiotherapist for rehabilitation twice per week, instead of thrice per week, i.e. recovery duration reduces if the patient goes three times for rehabilitation instead of two times.

Step 1: Look for the problem in the hypothesis. The hypothesis either be a word or can be a statement. In the above example the hypothesis is:

“The expected recovery period in knee rehabilitation is more than 8 weeks”

Step 2: Make a mathematical statement from the hypothesis. Averages can also be represented as μ, thus the null hypothesis formula will be.

In the above equation, the hypothesis is equivalent to H1, the average is denoted by μ and > that the average is greater than eight.

Step 3: Explain what will come up if the hypothesis does not come right i.e., the rehabilitation period may not proceed more than 08 weeks.

There are two options: either the recovery will be less than or equal to 8 weeks.

H 0 : μ ≤ 8

In the above equation, the null hypothesis is equivalent to H 0 , the average is denoted by μ and ≤ represents that the average is less than or equal to eight.

What will happen if the scientist does not have any knowledge about the outcome?

Problem: An investigator investigates the post-operative impact and influence of radical exercise on patients who have operative procedures of the knee. The chances are either the exercise will improve the recovery or will make it worse. The usual time for recovery is 8 weeks.

Step 1: Make a null hypothesis i.e. the exercise does not show any effect and the recovery time remains almost 8 weeks.

H 0 : μ = 8

In the above equation, the null hypothesis is equivalent to H 0 , the average is denoted by μ, and the equal sign (=) shows that the average is equal to eight.

Step 2: Make the alternate hypothesis which is the reverse of the null hypothesis. Particularly what will happen if treatment (exercise) makes an impact?

In the above equation, the alternate hypothesis is equivalent to H1, the average is denoted by μ and not equal sign (≠) represents that the average is not equal to eight.

Significance Tests

To get a reasonable and probable clarification of statistics (data), a significance test is performed. The null hypothesis does not have data. It is a piece of information or statement which contains numerical figures about the population. The data can be in different forms like in means or proportions. It can either be the difference of proportions and means or any odd ratio.

The following table will explain the symbols:

P-value
Probability of success
Size of sample
Null Hypothesis
Alternate Hypothesis

P-value is the chief statistical final result of the significance test of the null hypothesis.

  • P-value = Pr(data or data more extreme | H 0 true)
  • | = “given”
  • Pr = probability
  • H 0 = the null hypothesis

The first stage of Null Hypothesis Significance Testing (NHST) is to form an alternate and null hypothesis. By this, the research question can be briefly explained.

Null Hypothesis = no effect of treatment, no difference, no association Alternative Hypothesis = effective treatment, difference, association

When to reject the null hypothesis?

Researchers will reject the null hypothesis if it is proven wrong after experimentation. Researchers accept null hypothesis to be true and correct until it is proven wrong or false. On the other hand, the researchers try to strengthen the alternate hypothesis. The binomial test is performed on a sample and after that, a series of tests were performed (Frick, 1995).

Step 1: Evaluate and read the research question carefully and consciously and make a null hypothesis. Verify the sample that supports the binomial proportion. If there is no difference then find out the value of the binomial parameter.

Show the null hypothesis as:

H 0 :p= the value of p if H 0 is true

To find out how much it varies from the proposed data and the value of the null hypothesis, calculate the sample proportion.

Step 2: In test statistics, find the binomial test that comes under the null hypothesis. The test must be based on precise and thorough probabilities. Also make a list of pmf that apply, when the null hypothesis proves true and correct.

When H 0 is true, X~b(n, p)

N = size of the sample

P = assume value if H 0 proves true.

Step 3: Find out the value of P. P-value is the probability of data that is under observation.

Rise or increase in the P value = Pr(X ≥ x)

X = observed number of successes

P value = Pr(X ≤ x).

Step 4: Demonstrate the findings or outcomes in a descriptive detailed way.

  • Sample proportion
  • The direction of difference (either increases or decreases)

Perceived Problems With the Null Hypothesis

Variable or model selection and less information in some cases are the chief important issues that affect the testing of the null hypothesis. Statistical tests of the null hypothesis are reasonably not strong. There is randomization about significance. (Gill, 1999) The main issue with the testing of the null hypothesis is that they all are wrong or false on a ground basis.

There is another problem with the a-level . This is an ignored but also a well-known problem. The value of a-level is without a theoretical basis and thus there is randomization in conventional values, most commonly 0.q, 0.5, or 0.01. If a fixed value of a is used, it will result in the formation of two categories (significant and non-significant) The issue of a randomized rejection or non-rejection is also present when there is a practical matter which is the strong point of the evidence related to a scientific matter.

The P-value has the foremost importance in the testing of null hypothesis but as an inferential tool and for interpretation, it has a problem. The P-value is the probability of getting a test statistic at least as extreme as the observed one.

The main point about the definition is: Observed results are not based on a-value

Moreover, the evidence against the null hypothesis was overstated due to unobserved results. A-value has importance more than just being a statement. It is a precise statement about the evidence from the observed results or data. Similarly, researchers found that P-values are objectionable. They do not prefer null hypotheses in testing. It is also clear that the P-value is strictly dependent on the null hypothesis. It is computer-based statistics. In some precise experiments, the null hypothesis statistics and actual sampling distribution are closely related but this does not become possible in observational studies.

Some researchers pointed out that the P-value is depending on the sample size. If the true and exact difference is small, a null hypothesis even of a large sample may get rejected. This shows the difference between biological importance and statistical significance. (Killeen, 2005)

Another issue is the fix a-level, i.e., 0.1. On the basis, if a-level a null hypothesis of a large sample may get accepted or rejected. If the size of simple is infinity and the null hypothesis is proved true there are still chances of Type I error. That is the reason this approach or method is not considered consistent and reliable. There is also another problem that the exact information about the precision and size of the estimated effect cannot be known. The only solution is to state the size of the effect and its precision.

Null Hypothesis Examples

Here are some examples:

Example 1: Hypotheses with One Sample of One Categorical Variable

Among all the population of humans, almost 10% of people prefer to do their task with their left hand i.e. left-handed. Let suppose, a researcher in the Penn States says that the population of students at the College of Arts and Architecture is mostly left-handed as compared to the general population of humans in general public society. In this case, there is only a sample and there is a comparison among the known population values to the population proportion of sample value.

  • Research Question: Do artists more expected to be left-handed as compared to the common population persons in society?
  • Response Variable: Sorting the student into two categories. One category has left-handed persons and the other category have right-handed persons.
  • Form Null Hypothesis: Arts and Architecture college students are no more predicted to be lefty as compared to the common population persons in society (Lefty students of Arts and Architecture college population is 10% or p= 0.10)

Example 2: Hypotheses with One Sample of One Measurement Variable

A generic brand of antihistamine Diphenhydramine making medicine in the form of a capsule, having a 50mg dose. The maker of the medicines is concerned that the machine has come out of calibration and is not making more capsules with the suitable and appropriate dose.

  • Research Question: Does the statistical data recommended about the mean and average dosage of the population differ from 50mg?
  • Response Variable: Chemical assay used to find the appropriate dosage of the active ingredient.
  • Null Hypothesis: Usually, the 50mg dosage of capsules of this trade name (population average and means dosage =50 mg).

Example 3: Hypotheses with Two Samples of One Categorical Variable

Several people choose vegetarian meals on a daily basis. Typically, the researcher thought that females like vegetarian meals more than males.

  • Research Question: Does the data recommend that females (women) prefer vegetarian meals more than males (men) regularly?
  • Response Variable: Cataloguing the persons into vegetarian and non-vegetarian categories. Grouping Variable: Gender
  • Null Hypothesis: Gender is not linked to those who like vegetarian meals. (Population percent of women who eat vegetarian meals regularly = population percent of men who eat vegetarian meals regularly or p women = p men).

Example 4: Hypotheses with Two Samples of One Measurement Variable

Nowadays obesity and being overweight is one of the major and dangerous health issues. Research is performed to confirm that a low carbohydrates diet leads to faster weight loss than a low-fat diet.

  • Research Question: Does the given data recommend that usually, a low-carbohydrate diet helps in losing weight faster as compared to a low-fat diet?
  • Response Variable: Weight loss (pounds)
  • Explanatory Variable: Form of diet either low carbohydrate or low fat
  • Null Hypothesis: There is no significant difference when comparing the mean loss of weight of people using a low carbohydrate diet to people using a diet having low fat. (population means loss of weight on a low carbohydrate diet = population means loss of weight on a diet containing low fat).

Example 5: Hypotheses about the relationship between Two Categorical Variables

A case-control study was performed. The study contains nonsmokers, stroke patients, and controls. The subjects are of the same occupation and age and the question was asked if someone at their home or close surrounding smokes?

  • Research Question: Did second-hand smoke enhance the chances of stroke?
  • Variables: There are 02 diverse categories of variables. (Controls and stroke patients) (whether the smoker lives in the same house). The chances of having a stroke will be increased if a person is living with a smoker.
  • Null Hypothesis: There is no significant relationship between a passive smoker and stroke or brain attack. (odds ratio between stroke and the passive smoker is equal to 1).

Example 6: Hypotheses about the relationship between Two Measurement Variables

A financial expert observes that there is somehow a positive and effective relationship between the variation in stock rate price and the quantity of stock bought by non-management employees

  • Response variable- Regular alteration in price
  • Explanatory Variable- Stock bought by non-management employees
  • Null Hypothesis: The association and relationship between the regular stock price alteration ($) and the daily stock-buying by non-management employees ($) = 0.

Example 7: Hypotheses about comparing the relationship between Two Measurement Variables in Two Samples

  • Research Question: Is the relation between the bill paid in a restaurant and the tip given to the waiter, is linear? Is this relation different for dining and family restaurants?
  • Explanatory Variable- total bill amount
  • Response Variable- the amount of tip
  • Null Hypothesis: The relationship and association between the total bill quantity at a family or dining restaurant and the tip, is the same.

Try to answer the quiz below to check what you have learned so far about the null hypothesis.

Choose the best answer. 

Send Your Results (Optional)

clock.png

  • Blackwelder, W. C. (1982). “Proving the null hypothesis” in clinical trials. Controlled Clinical Trials , 3(4), 345–353.
  • Frick, R. W. (1995). Accepting the null hypothesis. Memory & Cognition, 23(1), 132–138.
  • Gill, J. (1999). The insignificance of null hypothesis significance testing. Political Research Quarterly , 52(3), 647–674.
  • Killeen, P. R. (2005). An alternative to null-hypothesis significance tests. Psychological Science, 16(5), 345–353.

©BiologyOnline.com. Content provided and moderated by Biology Online Editors.

Last updated on June 16th, 2022

You will also like...

how to write a null hypothesis ap bio

Ecological Research: Measuring & Analysis

This lesson is about the methods used for ecological research, such as quadrat and transect sampling, canopy fogging, an..

Plant Auxins

Plant Auxins – Phototropism & Geotropism

Plants produce hormones to regulate their growth. Auxins, for instance, influence plant growth. Know the role of auxin i..

green plant cells

Plant Cells vs. Animal Cells

Plant cells have plastids essential in photosynthesis. They also have an additional layer called cell wall on their cell..

Cambial cells

Plant Tissues

Plant organs are comprised of tissues working together for a common function. The different types of plant tissues are m..

human brain structure

The Conscious & Unconscious Nervous System

This tutorial elaborates on how the nervous system works, particularly at the tissue level of the brain. There are three..

Early Mammals on Earth

Early Mammals on Earth

The Earth's ecosphere was rapidly changing and throwing up a wide range of ecological niches that new adaptive organisms..

Related Articles...

how to write a null hypothesis ap bio

No related articles found

Statology

Statistics Made Easy

How to Write a Null Hypothesis (5 Examples)

A hypothesis test uses sample data to determine whether or not some claim about a population parameter is true.

Whenever we perform a hypothesis test, we always write a null hypothesis and an alternative hypothesis, which take the following forms:

H 0 (Null Hypothesis): Population parameter =,  ≤, ≥ some value

H A  (Alternative Hypothesis): Population parameter <, >, ≠ some value

Note that the null hypothesis always contains the equal sign .

We interpret the hypotheses as follows:

Null hypothesis: The sample data provides no evidence to support some claim being made by an individual.

Alternative hypothesis: The sample data  does provide sufficient evidence to support the claim being made by an individual.

For example, suppose it’s assumed that the average height of a certain species of plant is 20 inches tall. However, one botanist claims the true average height is greater than 20 inches.

To test this claim, she may go out and collect a random sample of plants. She can then use this sample data to perform a hypothesis test using the following two hypotheses:

H 0 : μ ≤ 20 (the true mean height of plants is equal to or even less than 20 inches)

H A : μ > 20 (the true mean height of plants is greater than 20 inches)

If the sample data gathered by the botanist shows that the mean height of this species of plants is significantly greater than 20 inches, she can reject the null hypothesis and conclude that the mean height is greater than 20 inches.

Read through the following examples to gain a better understanding of how to write a null hypothesis in different situations.

Example 1: Weight of Turtles

A biologist wants to test whether or not the true mean weight of a certain species of turtles is 300 pounds. To test this, he goes out and measures the weight of a random sample of 40 turtles.

Here is how to write the null and alternative hypotheses for this scenario:

H 0 : μ = 300 (the true mean weight is equal to 300 pounds)

H A : μ ≠ 300 (the true mean weight is not equal to 300 pounds)

Example 2: Height of Males

It’s assumed that the mean height of males in a certain city is 68 inches. However, an independent researcher believes the true mean height is greater than 68 inches. To test this, he goes out and collects the height of 50 males in the city.

H 0 : μ ≤ 68 (the true mean height is equal to or even less than 68 inches)

H A : μ > 68 (the true mean height is greater than 68 inches)

Example 3: Graduation Rates

A university states that 80% of all students graduate on time. However, an independent researcher believes that less than 80% of all students graduate on time. To test this, she collects data on the proportion of students who graduated on time last year at the university.

H 0 : p ≥ 0.80 (the true proportion of students who graduate on time is 80% or higher)

H A : μ < 0.80 (the true proportion of students who graduate on time is less than 80%)

Example 4: Burger Weights

A food researcher wants to test whether or not the true mean weight of a burger at a certain restaurant is 7 ounces. To test this, he goes out and measures the weight of a random sample of 20 burgers from this restaurant.

H 0 : μ = 7 (the true mean weight is equal to 7 ounces)

H A : μ ≠ 7 (the true mean weight is not equal to 7 ounces)

Example 5: Citizen Support

A politician claims that less than 30% of citizens in a certain town support a certain law. To test this, he goes out and surveys 200 citizens on whether or not they support the law.

H 0 : p ≥ .30 (the true proportion of citizens who support the law is greater than or equal to 30%)

H A : μ < 0.30 (the true proportion of citizens who support the law is less than 30%)

Additional Resources

Introduction to Hypothesis Testing Introduction to Confidence Intervals An Explanation of P-Values and Statistical Significance

Featured Posts

how to write a null hypothesis ap bio

Hey there. My name is Zach Bobbitt. I have a Masters of Science degree in Applied Statistics and I’ve worked on machine learning algorithms for professional businesses in both healthcare and retail. I’m passionate about statistics, machine learning, and data visualization and I created Statology to be a resource for both students and teachers alike.  My goal with this site is to help you learn statistics through using simple terms, plenty of real-world examples, and helpful illustrations.

2 Replies to “How to Write a Null Hypothesis (5 Examples)”

you are amazing, thank you so much

Say I am a botanist hypothesizing the average height of daisies is 20 inches, or not? Does T = (ave – 20 inches) / √ variance / (80 / 4)? … This assumes 40 real measures + 40 fake = 80 n, but that seems questionable. Please advise.

Leave a Reply Cancel reply

Your email address will not be published. Required fields are marked *

Join the Statology Community

Sign up to receive Statology's exclusive study resource: 100 practice problems with step-by-step solutions. Plus, get our latest insights, tutorials, and data analysis tips straight to your inbox!

By subscribing you accept Statology's Privacy Policy.

  • PRO Courses Guides New Tech Help Pro Expert Videos About wikiHow Pro Upgrade Sign In
  • EDIT Edit this Article
  • EXPLORE Tech Help Pro About Us Random Article Quizzes Request a New Article Community Dashboard This Or That Game Popular Categories Arts and Entertainment Artwork Books Movies Computers and Electronics Computers Phone Skills Technology Hacks Health Men's Health Mental Health Women's Health Relationships Dating Love Relationship Issues Hobbies and Crafts Crafts Drawing Games Education & Communication Communication Skills Personal Development Studying Personal Care and Style Fashion Hair Care Personal Hygiene Youth Personal Care School Stuff Dating All Categories Arts and Entertainment Finance and Business Home and Garden Relationship Quizzes Cars & Other Vehicles Food and Entertaining Personal Care and Style Sports and Fitness Computers and Electronics Health Pets and Animals Travel Education & Communication Hobbies and Crafts Philosophy and Religion Work World Family Life Holidays and Traditions Relationships Youth
  • Browse Articles
  • Learn Something New
  • Quizzes Hot
  • This Or That Game
  • Train Your Brain
  • Explore More
  • Support wikiHow
  • About wikiHow
  • Log in / Sign up
  • Education and Communications
  • College University and Postgraduate
  • Academic Writing

Writing Null Hypotheses in Research and Statistics

Last Updated: January 17, 2024 Fact Checked

This article was co-authored by Joseph Quinones and by wikiHow staff writer, Jennifer Mueller, JD . Joseph Quinones is a High School Physics Teacher working at South Bronx Community Charter High School. Joseph specializes in astronomy and astrophysics and is interested in science education and science outreach, currently practicing ways to make physics accessible to more students with the goal of bringing more students of color into the STEM fields. He has experience working on Astrophysics research projects at the Museum of Natural History (AMNH). Joseph recieved his Bachelor's degree in Physics from Lehman College and his Masters in Physics Education from City College of New York (CCNY). He is also a member of a network called New York City Men Teach. There are 7 references cited in this article, which can be found at the bottom of the page. This article has been fact-checked, ensuring the accuracy of any cited facts and confirming the authority of its sources. This article has been viewed 26,303 times.

Are you working on a research project and struggling with how to write a null hypothesis? Well, you've come to the right place! Start by recognizing that the basic definition of "null" is "none" or "zero"—that's your biggest clue as to what a null hypothesis should say. Keep reading to learn everything you need to know about the null hypothesis, including how it relates to your research question and your alternative hypothesis as well as how to use it in different types of studies.

Things You Should Know

  • Write a research null hypothesis as a statement that the studied variables have no relationship to each other, or that there's no difference between 2 groups.

{\displaystyle \mu _{1}=\mu _{2}}

  • Adjust the format of your null hypothesis to match the statistical method you used to test it, such as using "mean" if you're comparing the mean between 2 groups.

What is a null hypothesis?

A null hypothesis states that there's no relationship between 2 variables.

  • Research hypothesis: States in plain language that there's no relationship between the 2 variables or there's no difference between the 2 groups being studied.
  • Statistical hypothesis: States the predicted outcome of statistical analysis through a mathematical equation related to the statistical method you're using.

Examples of Null Hypotheses

Step 1 Research question:

Null Hypothesis vs. Alternative Hypothesis

Step 1 Null hypotheses and alternative hypotheses are mutually exclusive.

  • For example, your alternative hypothesis could state a positive correlation between 2 variables while your null hypothesis states there's no relationship. If there's a negative correlation, then both hypotheses are false.

Step 2 Proving the null hypothesis false is a precursor to proving the alternative.

  • You need additional data or evidence to show that your alternative hypothesis is correct—proving the null hypothesis false is just the first step.
  • In smaller studies, sometimes it's enough to show that there's some relationship and your hypothesis could be correct—you can leave the additional proof as an open question for other researchers to tackle.

How do I test a null hypothesis?

Use statistical methods on collected data to test the null hypothesis.

  • Group means: Compare the mean of the variable in your sample with the mean of the variable in the general population. [6] X Research source
  • Group proportions: Compare the proportion of the variable in your sample with the proportion of the variable in the general population. [7] X Research source
  • Correlation: Correlation analysis looks at the relationship between 2 variables—specifically, whether they tend to happen together. [8] X Research source
  • Regression: Regression analysis reveals the correlation between 2 variables while also controlling for the effect of other, interrelated variables. [9] X Research source

Templates for Null Hypotheses

Step 1 Group means

  • Research null hypothesis: There is no difference in the mean [dependent variable] between [group 1] and [group 2].

{\displaystyle \mu _{1}+\mu _{2}=0}

  • Research null hypothesis: The proportion of [dependent variable] in [group 1] and [group 2] is the same.

{\displaystyle p_{1}=p_{2}}

  • Research null hypothesis: There is no correlation between [independent variable] and [dependent variable] in the population.

\rho =0

  • Research null hypothesis: There is no relationship between [independent variable] and [dependent variable] in the population.

{\displaystyle \beta =0}

Expert Q&A

Joseph Quinones

You Might Also Like

Write an Essay

Expert Interview

how to write a null hypothesis ap bio

Thanks for reading our article! If you’d like to learn more about physics, check out our in-depth interview with Joseph Quinones .

  • ↑ https://online.stat.psu.edu/stat100/lesson/10/10.1
  • ↑ https://online.stat.psu.edu/stat501/lesson/2/2.12
  • ↑ https://support.minitab.com/en-us/minitab/21/help-and-how-to/statistics/basic-statistics/supporting-topics/basics/null-and-alternative-hypotheses/
  • ↑ https://www.ncbi.nlm.nih.gov/pmc/articles/PMC5635437/
  • ↑ https://online.stat.psu.edu/statprogram/reviews/statistical-concepts/hypothesis-testing
  • ↑ https://education.arcus.chop.edu/null-hypothesis-testing/
  • ↑ https://sphweb.bumc.bu.edu/otlt/mph-modules/bs/bs704_hypothesistest-means-proportions/bs704_hypothesistest-means-proportions_print.html

About This Article

Joseph Quinones

  • Send fan mail to authors

Reader Success Stories

Mogens Get

Dec 3, 2022

Did this article help you?

how to write a null hypothesis ap bio

Featured Articles

How to Get a Fade that Complements Your Style, Hair Type & More

Trending Articles

18 Practical Ways to Celebrate Pride as an Ally

Watch Articles

Clean Silver Jewelry with Vinegar

  • Terms of Use
  • Privacy Policy
  • Do Not Sell or Share My Info
  • Not Selling Info

wikiHow Tech Help Pro:

Level up your tech skills and stay ahead of the curve

AP Biology Exam Tips

The following strategies for answering the free-response questions will help you on exam day.

  • Before beginning to solve the free-response questions, it is a good idea to read through all the questions to determine which ones you feel most prepared to answer. You can then proceed to solve the questions in a sequence that will allow you to perform your best.
  • Monitor your time appropriately on the free-response section. You want to ensure that you do not spend too much time on one question that you do not have enough time to at least attempt to answer all of them.
  • Show all the steps you took to reach your solution on questions involving calculations. If you do work that you think is incorrect, simply put an "X" through it, instead of spending time erasing it completely.
  • Many free-response questions are divided into parts such as a, b, c, and d, with each part calling for a different response. Credit for each part is awarded independently, so you should attempt to solve each part. For example, you may receive no credit for your answer to part a, but still receive full credit for part b, c, or d. If the answer to a later part of a question depends on the answer to an earlier part, you may still be able to receive full credit for the later part, even if that earlier answer is wrong.
  • Organize your answers as clearly and neatly as possible. You might want to label your answers according to the sub-part, such as (a), (b), (c), etc. This will assist you in organizing your thoughts, as well as helping to ensure that you answer all the parts of the free-response question.
  • You should include the proper units for each number where appropriate. If you keep track of units as you perform your calculations, it can help ensure that you express answers in terms of the proper units. Depending on the exam question, it is often possible to lose points if the units are wrong or are missing from the answer.
  • You should not use the "scattershot" or “laundry list” approach: i.e., write as many equations or lists of terms as you can, hoping that the correct one will be among them so that you can get partial credit. For exams that ask for TWO or THREE examples or equations, only the first two or three examples will be scored.
  • Be sure to clearly and correctly label all graphs and diagrams accordingly. Read the question carefully, as this could include a graph title, x and y axes labels including units, a best fit line, etc.

Pay close attention to the task verbs used in the free-response questions. Each one directs you to complete a specific type of response. Here are the task verbs you’ll see on the exam:

  • Calculate: Perform mathematical steps to arrive at a final answer, including algebraic expressions, properly substituted numbers, and correct labeling of units and significant figures.
  • Construct/Draw: Create a diagram, graph, representation, or model that illustrates or explains relationships or phenomena. Labels may or may not be required.
  • Describe: Provide relevant characteristics of a specified topic.
  • Determine: Decide or conclude after reasoning, observation, or applying mathematical routines (calculations).
  • Evaluate: Judge or determine the significance or importance of information, or the quality or accuracy of a claim.
  • Explain: Provide information about how or why a relationship, process, pattern, position, situation, or outcome occurs, using evidence and/or reasoning to support or qualfiy a claim. Explain “how” typically requires analyzing the relationship, process, pattern, position, situation, or outcome; whereas, explain “why” typically requires analysis of motivations or reasons for the relationship, process, pattern, position, situation, or outcome.
  • Identify: Indicate or provide information about a specified topic, without elaboration or explanation.
  • Justify: Provide evidence to support, qualify, or defend a claim, and/or provide reasoning to explain how that evidence supports or qualifies the claim.
  • Make a claim: Make an assertion that is based on evidence or knowledge.
  • Predict/Make a prediction: Predict the causes or effects of a change in, or disruption to, one or more components in a relationship, pattern, process, or system.
  • Represent: Use appropriate graphs, symbols, words, illustrations, and/or tables of numerical values to describe biological concepts, characteristics, and/or relationships
  • State (the null/alternative hypothesis): Indicate or provide a hypothesis to support or defend a claim about a scientifically testable question.
  • Support a claim: Provide reasoning to explain how evidence supports or qualifies a claim.

Null Hypothesis Examples

ThoughtCo / Hilary Allison

  • Scientific Method
  • Chemical Laws
  • Periodic Table
  • Projects & Experiments
  • Biochemistry
  • Physical Chemistry
  • Medical Chemistry
  • Chemistry In Everyday Life
  • Famous Chemists
  • Activities for Kids
  • Abbreviations & Acronyms
  • Weather & Climate
  • Ph.D., Biomedical Sciences, University of Tennessee at Knoxville
  • B.A., Physics and Mathematics, Hastings College

In statistical analysis, the null hypothesis assumes there is no meaningful relationship between two variables. Testing the null hypothesis can tell you whether your results are due to the effect of manipulating ​a dependent variable or due to chance. It's often used in conjunction with an alternative hypothesis, which assumes there is, in fact, a relationship between two variables.

The null hypothesis is among the easiest hypothesis to test using statistical analysis, making it perhaps the most valuable hypothesis for the scientific method. By evaluating a null hypothesis in addition to another hypothesis, researchers can support their conclusions with a higher level of confidence. Below are examples of how you might formulate a null hypothesis to fit certain questions.

What Is the Null Hypothesis?

The null hypothesis states there is no relationship between the measured phenomenon (the dependent variable ) and the independent variable , which is the variable an experimenter typically controls or changes. You do not​ need to believe that the null hypothesis is true to test it. On the contrary, you will likely suspect there is a relationship between a set of variables. One way to prove that this is the case is to reject the null hypothesis. Rejecting a hypothesis does not mean an experiment was "bad" or that it didn't produce results. In fact, it is often one of the first steps toward further inquiry.

To distinguish it from other hypotheses , the null hypothesis is written as ​ H 0  (which is read as “H-nought,” "H-null," or "H-zero"). A significance test is used to determine the likelihood that the results supporting the null hypothesis are not due to chance. A confidence level of 95% or 99% is common. Keep in mind, even if the confidence level is high, there is still a small chance the null hypothesis is not true, perhaps because the experimenter did not account for a critical factor or because of chance. This is one reason why it's important to repeat experiments.

Examples of the Null Hypothesis

To write a null hypothesis, first start by asking a question. Rephrase that question in a form that assumes no relationship between the variables. In other words, assume a treatment has no effect. Write your hypothesis in a way that reflects this.

Are teens better at math than adults? Age has no effect on mathematical ability.
Does taking aspirin every day reduce the chance of having a heart attack? Taking aspirin daily does not affect heart attack risk.
Do teens use cell phones to access the internet more than adults? Age has no effect on how cell phones are used for internet access.
Do cats care about the color of their food? Cats express no food preference based on color.
Does chewing willow bark relieve pain? There is no difference in pain relief after chewing willow bark versus taking a placebo.

Other Types of Hypotheses

In addition to the null hypothesis, the alternative hypothesis is also a staple in traditional significance tests . It's essentially the opposite of the null hypothesis because it assumes the claim in question is true. For the first item in the table above, for example, an alternative hypothesis might be "Age does have an effect on mathematical ability."

Key Takeaways

  • In hypothesis testing, the null hypothesis assumes no relationship between two variables, providing a baseline for statistical analysis.
  • Rejecting the null hypothesis suggests there is evidence of a relationship between variables.
  • By formulating a null hypothesis, researchers can systematically test assumptions and draw more reliable conclusions from their experiments.
  • Difference Between Independent and Dependent Variables
  • Examples of Independent and Dependent Variables
  • What Is a Hypothesis? (Science)
  • What 'Fail to Reject' Means in a Hypothesis Test
  • Definition of a Hypothesis
  • Null Hypothesis Definition and Examples
  • Scientific Method Vocabulary Terms
  • Null Hypothesis and Alternative Hypothesis
  • Hypothesis Test for the Difference of Two Population Proportions
  • How to Conduct a Hypothesis Test
  • What Is a P-Value?
  • What Are the Elements of a Good Hypothesis?
  • Hypothesis Test Example
  • What Is the Difference Between Alpha and P-Values?
  • Understanding Path Analysis
  • An Example of a Hypothesis Test

AP Biology First Assessment

Profile Picture

Students also viewed

Profile Picture

Excel at Science

  • May 9, 2020

How to Answer Experiment Questions on AP Biology FRQ

Updated: Sep 22, 2023

On the AP Biology exam, the first section is multiple-choice and the second section is a set of 8 FRQs (free response questions), in which you may be given an experiment setup or asked to design an experiment yourself. Many students find the FRQs challenging because experimental design is not a specific chapter in the AP Biology textbook.

In order to answer these questions well, you need to put on your scientist’s hat and think about it as if you were running the experiment. The best way to demonstrate this is to walk through some examples of experiments. First, we will discuss the guidelines and terminology used for designing and running experiments in biology.

An experiment should always be based on a hypothesis, something that you believe might be true and that you want to test. If there is no hypothesis, there is no purpose for the experiment. Often, the hypothesis is an association between a factor and a result of interest. Some examples are:

Sunlight and plant growth

Mutation in bacteria and resistance to an antibiotic

A particular drug and decreased blood pressure

Soil acidity and flower color

Let’s take the second example, a particular mutation in bacteria and resistance to a specific antibiotic. There are so many different aspects of bacteria and the environment they live in. How can we determine that one particular trait (in this case, a mutated gene) is responsible for antibiotic resistance?

This is why scientists use controlled environments for their experiments. They can control for all factors ( keep them the same) across all experimental groups except the suspected factor, the gene mutation. Each experimental group has a different treatment or condition. In a control group, there is no special treatment. The control group serves as a baseline to compare the other groups to. The diagram below illustrates this:

Notice that all other factors (bacterial strain, concentration of nutrients, concentration of antibiotic added, etc.) are kept the same.

Another term often used in experimentation is null hypothesis . This is different from the scientific hypothesis! Many students get confused by that. The null hypothesis is more of a statistics term and it states that there will be no significant difference observed among the different experimental groups. Scientists usually hope to reject the null hypothesis , which means they do observe a real difference, supporting their scientific hypothesis . This will all become more clear when we walk through some examples.

Example Problems:

2017 FRQ - #2 Bees and Caffeine Experiment

This question involves an experiment about bees and the nectar they encounter while pollinating flowers. The scientists want to understand the role of caffeine on the bees’ memory.

The question gives a table showing the results of the experiment, shown below. It includes a control group and test group (caffeine). It also shows the probability of the bees returning to a recently visited nectar source. This probability is used to represent the bees’ short-term and long-term memory.

how to write a null hypothesis ap bio

As you read through the question and think about the experiment, you should consider the set of questions below. Just consider them, no need to write them down. They will help you plan out your responses to the actual problem:

What are these scientists testing in this experiment? In other words, what is their scientific hypothesis ?

What is the independent and dependent variable?

What is the difference between the control and test group? What’s the purpose of the control group? Note that sometimes there is more than one test group. Here, we only have one, which is the caffeine treatment group.

What is the null hypothesis ?

How could the experimental data be represented graphically?

What do the +/- values mean in each of the data cells?

If you are able to answer all those questions, you will have no trouble with this problem. So let’s answer them:

The scientific hypothesis is that exposure to caffeine is associated with the bees’ memory.

The independent variable is the treatment, which is exposure to caffeine. The dependent variable is what is impacted. Here, that is the bees’ memory.

The control group is exposed to no caffeine, while the treatment group is exposed to caffeine in the nectar. The control group serves as a baseline to compare the treatment group to. If we hypothesize that caffeine has a negative impact on memory, then the probability of revisiting the nectar source should be higher for the treatment compared to the control.

The null hypothesis states that there is no significant difference in memory between the control and treatment groups. Any difference observed would be due to chance. To support the scientific hypothesis, scientists need the data to reject the null hypothesis.

The data here should be represented by a bar graph. There will be two bars, one for control and one for treatment. There should also be error bars because the standard errors are included in the data. The graph would look something like this:

how to write a null hypothesis ap bio

2019 FRQ - #2 Ecological Relationship Between Two Protists

This question is about an experiment that investigates the ecological relationship between two protists. Are they competing for the same food? Does one predate on the other? Or do they live together in harmony and use different resources? That is what the scientists want to know.

The data collected in the experiment is given in the question, shown below.

how to write a null hypothesis ap bio

Let’s answer the same list of questions again to really understand the experiment.

The scientific question being tested is: what kind of ecological relationship do protist species A and B have?

The independent variable is the treatment, which is the two species living together. The dependent variable is the population size of each species over time.

The control group is the species grown separately. The test group is the species grown together.

The null hypothesis states that there is no significant difference in population size between the control and treatment groups at each time point. Any difference observed would be due to chance.

The data here should be represented by a line graph, since we have time as a factor. Time should be on the x-axis -- this is almost always the case. There will be two lines, one for control and one for treatment. The graph would look something like this:

how to write a null hypothesis ap bio

Want to improve your AP Bio free response scores, fast?

Check out the AP Bio Practice Portal , which is our popular vault of 300+ AP-style MCQ and FRQ problem sets with answers and explanations for every question. Don't waste any more time Googling practice problems or answers - try it out now!

Try the Practice Portal >

Recent posts.

How to Study for AP Biology Finals: Tactical Strategies for Success

How to Interpret Diagrams and Graphs on AP Biology Exams

How to Get a 5 on the AP Biology Exam: A Comprehensive Study Guide

PrepScholar

Choose Your Test

Sat / act prep online guides and tips, 4 top tips to make ap biology frqs a breeze.

author image

Advanced Placement (AP)

feature_apbio-cc0

AP Biology is known for being one of the tougher AP exams , and, for most students, the free-response section is the hardest part of the test. In 2021 , the average score for every free-response question was less than a 50%! However, knowing what to expect can make it easier to get a great score on AP Biology FRQ. And in this guide, we explain everything you need to know to ace this section. Read on to learn the format of AP Biology FRQ, what graders are looking for, what the questions will look like, and what you can do to be well-prepared on exam day.

What's the Format of the AP Biology Free Response Section?

The AP Biology exam has two sections: multiple choice and free response. The free-section comes second and contains six questions:

  • Two long-response questions , both with a focus on analyzing experimental results. The second long question will require you to create a graph.
  • Four short-answer questions on the following topics in this order:
  • Scientific Investigation
  • Conceptual Analysis
  • Analysis of Model or Visual Representation
  • Analysis of Data

Additionally:

  • The free-response section is 90 minutes long
  • It's worth 50% of your total score
  • You're able to use the AP Biology formula sheet for the entire section

Long questions are worth 8-10 points each, whereas short-answer questions are each worth 4 points. It's recommended that you spend about 25 minutes on each long question and about 10 minutes on each short question (although you'll decide yourself how long you spend on each question).

The AP Biology test expects you to know how to:

  • Understand how graphical and mathematical models can be used to explain biological principles and concepts
  • Make predictions and justify events based on biological principles
  • Implement your knowledge of proper experimental design
  • Interpret data

AP Biology Sample Free Response Questions 

Now we'll go through two AP Biology free response example questions: one long question and one short question. These questions both were used for the 2021 AP Biology exam . You can see answers and scoring for each of the 2021 AP Biology FRQs here .

Long Question

First let's look at one of the long questions. This is Question 2, so remember you'll need to create a graph for at least one part of it. The entire question is worth 8 points.

long1

Part A (1 point)

Part B (4 points)

First, you need to create a graph based on the data in Table 1. The graph is worth 3 points: 1 for axis labels, 1 for the correct plotting in the bar graph, and 1 for the error bars. Here's an example of a graph that would get full points:

1graph

Part C (1 point)

Part D (2 points)

  • The data do not support the claim because females III-2 and III-6 have the disorder and, if inheritance was X-linked recessive, they'd only have the disorder if their father II-1 had the disorder, which he does not.
  • The data supports mitochondrial inheritance because all of the offspring of individual II-2 , not just the sons, have the disorder.

Giving one of those answers is worth one point.

body_student_chemistry_major_microscope

Short Question

Next is a short question. It's question 3 in the free-response section which means it will focus on scientific investigation. It's worth a total of four points.

short1

Part B (1 point)

  • The researchers must run the experiment without adding resveratrol.
  • The researchers must treat the cells with DMSO alone.

There are two potential answers; you only need to include one:

  • No ATP production
  • Reduced ATP production

Part D (1 point)

For Part D, you must state that more electrons can be transferred so that more oxygen is required as the final electron acceptor.

Where to Find AP Biology FRQs

Taking practice tests and answering practice questions is one of the best ways to prepare for any AP exam, including AP Biology FRQs. Fortunately, the College Board, who creates and administers AP courses and exams, has made dozens of old AP Bio FRQs available for free online. Because there are so many official FRQs available, we recommend only using them instead of looking online for unofficial questions (those not created by the College Board), which can be hit or miss in terms of quality. However, if you're using an AP Biology prep book, they often have solid FRQs. For advice on which prep book to get, check out our guide on the best AP Biology prep books.

Here are links to the FRQs:

Additionally, the AP Biology Course and Exam Description includes two up-to-date FRQs, beginning on page 206.

Note that, until 2020, the AP Bio exam had six short-answer questions instead of the current four. This means that questions from 2019 and earlier will have a different format and slightly different content. They can still be useful to study, but be aware of the differences between them and the current free-response section.

feature_dna

4 Tips for AP Biology FRQs

When you're studying for AP Bio FRQs and actually taking the exam, there are a lot of things to remember to ensure you do your absolute best. Keep these four tips in mind throughout the year and on exam day.

#1: Know Your 13 Required Labs

There are 13 labs you're required to complete during the AP Biology course. Questions that relate at least in part to these labs make up 25% of the AP Biology exam. It’s important to understand how these labs are conducted and how the principles behind them relate to the main ideas of the course. This will help in answering both free-response and multiple-choice questions that deal with lab scenarios on the test. There's a nice overview of each of the 13 labs on this site that can refresh your memory, and we link to in-depth explanations of each of the labs in our AP Biology study guide .

You should also know general lab skills. Many free-response questions ask you to identify the components of a proposed experiment (dependent and independent variables) or to design a lab to test a certain hypothesis. You might have forgotten about the labs you did toward the beginning of the year, so take extra care to go over them. Make sure that you understand just how they were conducted and what the results mean.

#2: Eliminate Irrelevant Information

Free-response AP Biology questions (especially the long questions) include lots of scientific terminology and visual aids, and this kind of format might be intimidating if you’re not used to it. It’s important to practice sorting through this jumble of information so that you can quickly get to the root of the question rather than obsessing over small details you don’t understand.

Try underlining important words and phrases in the question to help you stay focused on the main points and avoid misleading distractions.

You should also practice responding to free-response questions in a straightforward way without any unnecessary fluff. Remember, this isn’t an English test; the graders are just looking for clear facts and analysis. Make it easy for them to give you points!

#3: Draw During Studying

If you're feeling shaky on your knowledge of a process or system in AP Biology, one helpful strategy is to draw it. This will both reinforce what you know and highlight what you still need to work on learning. Once you're able to draw an accurate diagram of a system or process without looking at your notes, you can feel confident that you know exactly how it works.

For example, you could challenge yourself to draw a diagram of a cell membrane, label its different components, and explain their significance. You could also draw a process like mitosis that happens in clear visual stages, or a more complex process like cellular respiration where you might focus on one aspect at a time (glycolysis, Krebs cycle, electron transport chain). You can also apply this tip during the exam, if you need help visualizing part of an AP Bio FRQ.

#4: Pay Attention to the Clock

Time is always tight on AP exams. For the AP Biology free response section, you get 90 minutes to answer six questions. It can be easy to get caught up on one question and suddenly realize you're nearly out of time but haven't had a chance to look at some of the questions, let alone answer them. Don't let this happen to you! We recommend spending 25 minutes on each of the two long questions and 10 minutes on each of the four short questions. You don't need to keep perfectly to that plan, but don't get too far off it, either. 

At the very least, make note of where you are halfway through the free-response section (that's 45 minutes in). If you're roughly halfway finished with the section (taking into account that long questions take about twice as much time to complete as short questions), you're doing well. If you're significantly behind that, you know you need to pick up the pace.

Also, don't feel you need to answer the FRQ in the order they're listed. We recommend skimming through each of the questions at the start of the section, then tackling the questions that seem easiest first so you can spend more time on trickier questions.

Summary: Acing the AP Biology Free Response Section

The AP Biology free-response section can be tough, but if you prepare well for it, you can go into exam day confident and knowing what to expect. The section consists of two long questions and four short questions, lasts 90 minutes, and is worth half of your total score. You'll need to create a graph for the second AP biology FRQ. Old exam questions are a great study resource and, when you're preparing for the free-response section, keep these four tips in mind:

  • Know your labs
  • Eliminate irrelevant information
  • Make drawings while studying
  • Stay aware of time

What's Next?

How should you study for the AP Biology exam? Our expert article goes over all 5 steps to take during your AP Biology review.

What is the rest of the AP Biology exam like? Our article on the AP Biology exam goes over every question type you can expect to see as well as tips for answering them.

Looking for an easier AP class than Biology? Learn which AP classes tend to be the least challenging for students .

Looking for help studying for your AP exam? Our one-on-one online AP tutoring services can help you prepare for your AP exams. Get matched with a top tutor who got a high score on the exam you're studying for!

Christine graduated from Michigan State University with degrees in Environmental Biology and Geography and received her Master's from Duke University. In high school she scored in the 99th percentile on the SAT and was named a National Merit Finalist. She has taught English and biology in several countries.

Ask a Question Below

Have any questions about this article or other topics? Ask below and we'll reply!

Improve With Our Famous Guides

  • For All Students

The 5 Strategies You Must Be Using to Improve 160+ SAT Points

How to Get a Perfect 1600, by a Perfect Scorer

Series: How to Get 800 on Each SAT Section:

Score 800 on SAT Math

Score 800 on SAT Reading

Score 800 on SAT Writing

Series: How to Get to 600 on Each SAT Section:

Score 600 on SAT Math

Score 600 on SAT Reading

Score 600 on SAT Writing

Free Complete Official SAT Practice Tests

What SAT Target Score Should You Be Aiming For?

15 Strategies to Improve Your SAT Essay

The 5 Strategies You Must Be Using to Improve 4+ ACT Points

How to Get a Perfect 36 ACT, by a Perfect Scorer

Series: How to Get 36 on Each ACT Section:

36 on ACT English

36 on ACT Math

36 on ACT Reading

36 on ACT Science

Series: How to Get to 24 on Each ACT Section:

24 on ACT English

24 on ACT Math

24 on ACT Reading

24 on ACT Science

What ACT target score should you be aiming for?

ACT Vocabulary You Must Know

ACT Writing: 15 Tips to Raise Your Essay Score

How to Get Into Harvard and the Ivy League

How to Get a Perfect 4.0 GPA

How to Write an Amazing College Essay

What Exactly Are Colleges Looking For?

Is the ACT easier than the SAT? A Comprehensive Guide

Should you retake your SAT or ACT?

When should you take the SAT or ACT?

Stay Informed

Follow us on Facebook (icon)

Get the latest articles and test prep tips!

Looking for Graduate School Test Prep?

Check out our top-rated graduate blogs here:

GRE Online Prep Blog

GMAT Online Prep Blog

TOEFL Online Prep Blog

Holly R. "I am absolutely overjoyed and cannot thank you enough for helping me!”

Have a language expert improve your writing

Run a free plagiarism check in 10 minutes, generate accurate citations for free.

  • Knowledge Base
  • Null and Alternative Hypotheses | Definitions & Examples

Null & Alternative Hypotheses | Definitions, Templates & Examples

Published on May 6, 2022 by Shaun Turney . Revised on June 22, 2023.

The null and alternative hypotheses are two competing claims that researchers weigh evidence for and against using a statistical test :

  • Null hypothesis ( H 0 ): There’s no effect in the population .
  • Alternative hypothesis ( H a or H 1 ) : There’s an effect in the population.

Table of contents

Answering your research question with hypotheses, what is a null hypothesis, what is an alternative hypothesis, similarities and differences between null and alternative hypotheses, how to write null and alternative hypotheses, other interesting articles, frequently asked questions.

The null and alternative hypotheses offer competing answers to your research question . When the research question asks “Does the independent variable affect the dependent variable?”:

  • The null hypothesis ( H 0 ) answers “No, there’s no effect in the population.”
  • The alternative hypothesis ( H a ) answers “Yes, there is an effect in the population.”

The null and alternative are always claims about the population. That’s because the goal of hypothesis testing is to make inferences about a population based on a sample . Often, we infer whether there’s an effect in the population by looking at differences between groups or relationships between variables in the sample. It’s critical for your research to write strong hypotheses .

You can use a statistical test to decide whether the evidence favors the null or alternative hypothesis. Each type of statistical test comes with a specific way of phrasing the null and alternative hypothesis. However, the hypotheses can also be phrased in a general way that applies to any test.

Here's why students love Scribbr's proofreading services

Discover proofreading & editing

The null hypothesis is the claim that there’s no effect in the population.

If the sample provides enough evidence against the claim that there’s no effect in the population ( p ≤ α), then we can reject the null hypothesis . Otherwise, we fail to reject the null hypothesis.

Although “fail to reject” may sound awkward, it’s the only wording that statisticians accept . Be careful not to say you “prove” or “accept” the null hypothesis.

Null hypotheses often include phrases such as “no effect,” “no difference,” or “no relationship.” When written in mathematical terms, they always include an equality (usually =, but sometimes ≥ or ≤).

You can never know with complete certainty whether there is an effect in the population. Some percentage of the time, your inference about the population will be incorrect. When you incorrectly reject the null hypothesis, it’s called a type I error . When you incorrectly fail to reject it, it’s a type II error.

Examples of null hypotheses

The table below gives examples of research questions and null hypotheses. There’s always more than one way to answer a research question, but these null hypotheses can help you get started.

( )
Does tooth flossing affect the number of cavities? Tooth flossing has on the number of cavities. test:

The mean number of cavities per person does not differ between the flossing group (µ ) and the non-flossing group (µ ) in the population; µ = µ .

Does the amount of text highlighted in the textbook affect exam scores? The amount of text highlighted in the textbook has on exam scores. :

There is no relationship between the amount of text highlighted and exam scores in the population; β = 0.

Does daily meditation decrease the incidence of depression? Daily meditation the incidence of depression.* test:

The proportion of people with depression in the daily-meditation group ( ) is greater than or equal to the no-meditation group ( ) in the population; ≥ .

*Note that some researchers prefer to always write the null hypothesis in terms of “no effect” and “=”. It would be fine to say that daily meditation has no effect on the incidence of depression and p 1 = p 2 .

The alternative hypothesis ( H a ) is the other answer to your research question . It claims that there’s an effect in the population.

Often, your alternative hypothesis is the same as your research hypothesis. In other words, it’s the claim that you expect or hope will be true.

The alternative hypothesis is the complement to the null hypothesis. Null and alternative hypotheses are exhaustive, meaning that together they cover every possible outcome. They are also mutually exclusive, meaning that only one can be true at a time.

Alternative hypotheses often include phrases such as “an effect,” “a difference,” or “a relationship.” When alternative hypotheses are written in mathematical terms, they always include an inequality (usually ≠, but sometimes < or >). As with null hypotheses, there are many acceptable ways to phrase an alternative hypothesis.

Examples of alternative hypotheses

The table below gives examples of research questions and alternative hypotheses to help you get started with formulating your own.

Does tooth flossing affect the number of cavities? Tooth flossing has an on the number of cavities. test:

The mean number of cavities per person differs between the flossing group (µ ) and the non-flossing group (µ ) in the population; µ ≠ µ .

Does the amount of text highlighted in a textbook affect exam scores? The amount of text highlighted in the textbook has an on exam scores. :

There is a relationship between the amount of text highlighted and exam scores in the population; β ≠ 0.

Does daily meditation decrease the incidence of depression? Daily meditation the incidence of depression. test:

The proportion of people with depression in the daily-meditation group ( ) is less than the no-meditation group ( ) in the population; < .

Null and alternative hypotheses are similar in some ways:

  • They’re both answers to the research question.
  • They both make claims about the population.
  • They’re both evaluated by statistical tests.

However, there are important differences between the two types of hypotheses, summarized in the following table.

A claim that there is in the population. A claim that there is in the population.

Equality symbol (=, ≥, or ≤) Inequality symbol (≠, <, or >)
Rejected Supported
Failed to reject Not supported

To help you write your hypotheses, you can use the template sentences below. If you know which statistical test you’re going to use, you can use the test-specific template sentences. Otherwise, you can use the general template sentences.

General template sentences

The only thing you need to know to use these general template sentences are your dependent and independent variables. To write your research question, null hypothesis, and alternative hypothesis, fill in the following sentences with your variables:

Does independent variable affect dependent variable ?

  • Null hypothesis ( H 0 ): Independent variable does not affect dependent variable.
  • Alternative hypothesis ( H a ): Independent variable affects dependent variable.

Test-specific template sentences

Once you know the statistical test you’ll be using, you can write your hypotheses in a more precise and mathematical way specific to the test you chose. The table below provides template sentences for common statistical tests.

( )
test 

with two groups

The mean dependent variable does not differ between group 1 (µ ) and group 2 (µ ) in the population; µ = µ . The mean dependent variable differs between group 1 (µ ) and group 2 (µ ) in the population; µ ≠ µ .
with three groups The mean dependent variable does not differ between group 1 (µ ), group 2 (µ ), and group 3 (µ ) in the population; µ = µ = µ . The mean dependent variable of group 1 (µ ), group 2 (µ ), and group 3 (µ ) are not all equal in the population.
There is no correlation between independent variable and dependent variable in the population; ρ = 0. There is a correlation between independent variable and dependent variable in the population; ρ ≠ 0.
There is no relationship between independent variable and dependent variable in the population; β = 0. There is a relationship between independent variable and dependent variable in the population; β ≠ 0.
Two-proportions test The dependent variable expressed as a proportion does not differ between group 1 ( ) and group 2 ( ) in the population; = . The dependent variable expressed as a proportion differs between group 1 ( ) and group 2 ( ) in the population; ≠ .

Note: The template sentences above assume that you’re performing one-tailed tests . One-tailed tests are appropriate for most studies.

If you want to know more about statistics , methodology , or research bias , make sure to check out some of our other articles with explanations and examples.

  • Normal distribution
  • Descriptive statistics
  • Measures of central tendency
  • Correlation coefficient

Methodology

  • Cluster sampling
  • Stratified sampling
  • Types of interviews
  • Cohort study
  • Thematic analysis

Research bias

  • Implicit bias
  • Cognitive bias
  • Survivorship bias
  • Availability heuristic
  • Nonresponse bias
  • Regression to the mean

Hypothesis testing is a formal procedure for investigating our ideas about the world using statistics. It is used by scientists to test specific predictions, called hypotheses , by calculating how likely it is that a pattern or relationship between variables could have arisen by chance.

Null and alternative hypotheses are used in statistical hypothesis testing . The null hypothesis of a test always predicts no effect or no relationship between variables, while the alternative hypothesis states your research prediction of an effect or relationship.

The null hypothesis is often abbreviated as H 0 . When the null hypothesis is written using mathematical symbols, it always includes an equality symbol (usually =, but sometimes ≥ or ≤).

The alternative hypothesis is often abbreviated as H a or H 1 . When the alternative hypothesis is written using mathematical symbols, it always includes an inequality symbol (usually ≠, but sometimes < or >).

A research hypothesis is your proposed answer to your research question. The research hypothesis usually includes an explanation (“ x affects y because …”).

A statistical hypothesis, on the other hand, is a mathematical statement about a population parameter. Statistical hypotheses always come in pairs: the null and alternative hypotheses . In a well-designed study , the statistical hypotheses correspond logically to the research hypothesis.

Cite this Scribbr article

If you want to cite this source, you can copy and paste the citation or click the “Cite this Scribbr article” button to automatically add the citation to our free Citation Generator.

Turney, S. (2023, June 22). Null & Alternative Hypotheses | Definitions, Templates & Examples. Scribbr. Retrieved June 9, 2024, from https://www.scribbr.com/statistics/null-and-alternative-hypotheses/

Is this article helpful?

Shaun Turney

Shaun Turney

Other students also liked, inferential statistics | an easy introduction & examples, hypothesis testing | a step-by-step guide with easy examples, type i & type ii errors | differences, examples, visualizations, what is your plagiarism score.

HIGH SCHOOL

  • ACT Tutoring
  • SAT Tutoring
  • PSAT Tutoring
  • ASPIRE Tutoring
  • SHSAT Tutoring
  • STAAR Tutoring

GRADUATE SCHOOL

  • MCAT Tutoring
  • GRE Tutoring
  • LSAT Tutoring
  • GMAT Tutoring
  • AIMS Tutoring
  • HSPT Tutoring
  • ISAT Tutoring
  • SSAT Tutoring

Search 50+ Tests

Loading Page

math tutoring

  • Elementary Math
  • Pre-Calculus
  • Trigonometry

science tutoring

Foreign languages.

  • Mandarin Chinese

elementary tutoring

  • Computer Science

Search 350+ Subjects

  • Video Overview
  • Tutor Selection Process
  • Online Tutoring
  • Mobile Tutoring
  • Instant Tutoring
  • How We Operate
  • Our Guarantee
  • Impact of Tutoring
  • Reviews & Testimonials
  • About Varsity Tutors

AP Statistics : How to establish a null hypothesis

Study concepts, example questions & explanations for ap statistics, all ap statistics resources, example questions, example question #1 : how to establish a null hypothesis.

Jimmy thinks that Josh cannot shoot more than 50 points on average in a game. Josh disputes this claim and tells Jimmy that he is going to play 10 games and prove him wrong. What is the null hypothesis? 

Josh cannot shoot more than 50 points.

Josh cannot play 10 games.

Josh cannot shoot less than 50 points.

Josh cannot shoot exactly 50 points.

The null hypothesis is what we intend to either reject or fail to reject using our sample data. In this case, the null hypothesis is that Josh cannot shoot more than 50 points on average, and Josh's performance in 10 games are the sample data we use to assess this hypothesis. 

A student is beginning an analysis to determine whether there is a relationship between temperatures and traffic accidents.  The student is trying to articulate a null hypothesis for the study.  Which of the following is an acceptable null hypothesis?

There is a positive relationship between temperatures and traffic accidents

There is a negative relationship between temperatures and traffic accidents

No variable can accurately predict whether traffic accidents will increase

Traffic accidents increase as temperatures decrease

There is no relationship between temperatures and frequency of traffic accidents

The null hypothesis is the default hypothesis and predicts that there is no relationship between the variables in question.  Each of the incorrect answer choices here either predicts a relationship between variables or makes a broad assertion that includes much more than the variables in question.

how to write a null hypothesis ap bio

Not enough information to make a decision.

Conditionally

The statistician has determined that she will only reject the null hypothesis if she has 95% confidence that there is a relationship between variables. 

To have this level of confidence, the statistician must obtain a p value of 0.05 or lower.

Therefore, she should not reject the null hypothesis since 0.1 is greater that 0.05.

The Environmental Protection Agency (EPA) wants to test the pollution level of the Colorado River. If the pollution level is too high, the water will be stopped from going into drinking water pipelines. The EPA randomly chooses different spots along the river to collect water samples from, and then tests the samples for their pollution levels. Which of the following decisions would result from the type I error?

Keeping the drinking water pipelines open when the pollution levels are higher than the allowed limit. 

Keeping the drinking water pipelines open when the pollution levels are within the allowed limit. 

Closing the drinking water pipelines for the river when the pollution levels are within the allowed limit. 

Closing the drinking water pipelines when the pollution levels are higher than the allowed limit. 

Closing the drinking water pipelines because of the endangered frog population. 

The hypotheses tested here are: 

how to write a null hypothesis ap bio

The type I error occurs when the null hypothesis is rejected even though it is actually true. In this case, the type I error would be deciding that the mean pollution levels are higher than the allowed limit and closing the drinking water pipelines. 

A study would like to determine whether meditation helps students improve focus time. They know that the average focus time of an American 4 th grader is 23 minutes. They then recruit 50 meditators and calculate their average focus time. What is the appropriate null hypothesis for this study?

how to write a null hypothesis ap bio

A researcher wants to determine whether there is a significant linear relationship between time spent meditating and time spent studying. What is the appropriate null hypothesis for this study?

how to write a null hypothesis ap bio

This question is about a linear regression between time spent meditating and time spent studying. Therefore, the hypothesis is regarding Beta1, the slope of the line. We are testing a non-directional or bi-directional claim that the relationship is significant . Therefore, the null hypothesis is that the relationship is not significant, meaning the slope of the line is equal to zero.

A researcher wants to compare 3 different treatments to see if any of the treatments affects study time. The three treatments studied are control group, a group given vitamins, and a group given a placebo.  They found that the average time spent studying with control students was 2 hours, with students given vitamins it was 3 hours, and with placebos students studied 5 hours. Which of the following is the correct null hypothesis?

how to write a null hypothesis ap bio

Because we are comparing more than 2 groups, we must use an ANOVA for this problem. For an ANOVA problem, the null hypothesis is that all of the groups’ means are the same.

Example Question #31 : Significance

A researcher wants to investigate the claim that taking vitamins will help a student study longer. First, the researcher collects 32 students who do not take vitamins and determines their time spent studying. Then, the 32 students are given a vitamin for 1 week. After 1 week of taking vitamins, students are again tested to determine their time spent studying. Which of the following is the correct null hypothesis?

how to write a null hypothesis ap bio

Because the same students are tested twice, this is a paired study, therefore we must use a hypothesis appropriate for a paired t-test.  The hypothesis for a paired t-test regards the average of the differences between before and after treatment, called MuD. We are testing the claim that vitamins increase study time, which would mean that study time for vitamin users would be greater than that of the control.  Therefore the null must include all other outcomes. The null hypothesis should state that the difference between before and after treatment is greater than or equal to zero.

For her school science project, Susy wants to determine whether the ants in her neighborhood have smaller colonies than average. Research tells her that the average Harvester colony has around 4,000 ants. She counts the number of ants in 5 colonies in her neighborhood and determines the average colony size to be 3,700 ants. What is the appropriate null hypothesis for her science project?

how to write a null hypothesis ap bio

Susy wants to know whether ants in her neighborhood have smaller colonies, so that will be her alternative hypothesis. Therefore her null hypothesis needs to cover all other outcomes, that the colony sizes are greater than or equal to average colony size of 4000 ants.

For his school science project, Timmy wants to determine whether the ants in his neighborhood have colonies that are sized differently than normal. His research shows that the average Harvester colony has around 4000 ants. He counts the number of ants in 5 colonies and determines that the average colony size is 3,700 ants. What is the appropriate null hypothesis for his science project?

how to write a null hypothesis ap bio

Timmy does not have a directional hypothesis, he only wants to know whether local ant colonies are different from average. Therefore he thinks the colonies could be bigger or smaller than average. This means his alternative hypothesis is that the ant colonies are NOT equal to the average colony size of 4000 ants. His null hypothesis must include all other outcomes, which in this case is that local ant colonies are equal to the average size of 4000 ants. 

Display vt optimized

Report an issue with this question

If you've found an issue with this question, please let us know. With the help of the community we can continue to improve our educational resources.

DMCA Complaint

If you believe that content available by means of the Website (as defined in our Terms of Service) infringes one or more of your copyrights, please notify us by providing a written notice (“Infringement Notice”) containing the information described below to the designated agent listed below. If Varsity Tutors takes action in response to an Infringement Notice, it will make a good faith attempt to contact the party that made such content available by means of the most recent email address, if any, provided by such party to Varsity Tutors.

Your Infringement Notice may be forwarded to the party that made the content available or to third parties such as ChillingEffects.org.

Please be advised that you will be liable for damages (including costs and attorneys’ fees) if you materially misrepresent that a product or activity is infringing your copyrights. Thus, if you are not sure content located on or linked-to by the Website infringes your copyright, you should consider first contacting an attorney.

Please follow these steps to file a notice:

You must include the following:

A physical or electronic signature of the copyright owner or a person authorized to act on their behalf; An identification of the copyright claimed to have been infringed; A description of the nature and exact location of the content that you claim to infringe your copyright, in \ sufficient detail to permit Varsity Tutors to find and positively identify that content; for example we require a link to the specific question (not just the name of the question) that contains the content and a description of which specific portion of the question – an image, a link, the text, etc – your complaint refers to; Your name, address, telephone number and email address; and A statement by you: (a) that you believe in good faith that the use of the content that you claim to infringe your copyright is not authorized by law, or by the copyright owner or such owner’s agent; (b) that all of the information contained in your Infringement Notice is accurate, and (c) under penalty of perjury, that you are either the copyright owner or a person authorized to act on their behalf.

Send your complaint to our designated agent at:

Charles Cohn Varsity Tutors LLC 101 S. Hanley Rd, Suite 300 St. Louis, MO 63105

Or fill out the form below:

Contact Information

Complaint details.

Learning Tools by Varsity Tutors

Email address:
Your name:
Feedback:

If you're seeing this message, it means we're having trouble loading external resources on our website.

If you're behind a web filter, please make sure that the domains *.kastatic.org and *.kasandbox.org are unblocked.

To log in and use all the features of Khan Academy, please enable JavaScript in your browser.

AP®︎/College Statistics

Course: ap®︎/college statistics   >   unit 10.

  • Idea behind hypothesis testing

Examples of null and alternative hypotheses

  • Writing null and alternative hypotheses
  • P-values and significance tests
  • Comparing P-values to different significance levels
  • Estimating a P-value from a simulation
  • Estimating P-values from simulations
  • Using P-values to make conclusions

how to write a null hypothesis ap bio

Want to join the conversation?

  • Upvote Button navigates to signup page
  • Downvote Button navigates to signup page
  • Flag Button navigates to signup page

Good Answer

Video transcript

COMMENTS

  1. AP Biology Exam Prep: Research Questions + Null and ...

    In this video, I begin discussing AP Biology Science Practice 3: Questions and Methods by explaining how questions and hypotheses are formed at the beginning...

  2. AP Bio: How to write a null hypothesis : r/APStudents

    Okay, well this is late, but if it matters, for bio, the null hypothesis states that any difference between a predicted value and a experimental value are due to chance. So suppose I cross Aa with Aa. I should get, according to math, 25% AA, 50% Aa, and 25% aa. Now, suppose, in the experiment, i actually get something like 27% AA, 49% Aa and 24 ...

  3. The Chi Square Test: AP® Biology Crash Course

    So, for this example, we will say that we failed to reject the null hypothesis. The best way to get better at these statistical questions is to practice. Next, we will go through a question using the Chi Square Test that you could see on your AP® Bio exam. AP® Biology Exam Question. This question was adapted from the 2013 AP® Biology exam.

  4. Hypothesis Testing for AP Bio: Null, Alternative, and 95% ...

    Lecture and Practice with Null Hypothesis, Alternative Hypothesis, and 95% Confidence Intervals for AP Bio. AP Biology.

  5. AP Biology: Chi-Square Example

    In this video, students will learn to:-Write a null hypothesis that pertains to the investigation; -Determine the degrees of freedom (df) for an investigatio...

  6. Null hypothesis

    Biology definition: A null hypothesis is an assumption or proposition where an observed difference between two samples of a statistical population is purely accidental and not due to systematic causes. It is the hypothesis to be investigated through statistical hypothesis testing so that when refuted indicates that the alternative hypothesis is true. . Thus, a null hypothesis is a hypothesis ...

  7. How to Write a Null Hypothesis (5 Examples)

    Whenever we perform a hypothesis test, we always write a null hypothesis and an alternative hypothesis, which take the following forms: H0 (Null Hypothesis): Population parameter =, ≤, ≥ some value. HA (Alternative Hypothesis): Population parameter <, >, ≠ some value. Note that the null hypothesis always contains the equal sign.

  8. AP BIOLOGY: Null hypothesis Flashcards

    Study with Quizlet and memorize flashcards containing terms like null hypothesis, alternate hypothesis, Chi-square test and more. ... AP Biology Evolution Vocabulary. Teacher 56 terms. pirolliv. Preview. Exam 2- Ch.9- stages in prenatal development . 7 terms. harleee_grant. Preview. Biology Unit 3 Chapter 1. 30 terms.

  9. PDF M and M Chi Square Analysis

    the null hypothesis (that chance alone can explain the difference) or so far apart that the null hypothesis must be rejected. Accept the null hypothesis Reject the null hypothesis . Degrees of Freedom Probability 0.90 0.50 0.25 0.10 0.05 0.01 1 . 0.016 0.46 1.32 2.71 3.84 6.64 . 2 . 0.21 1.39 2.77 4.61 5.99 9.21 . 3

  10. How to Write a Null Hypothesis (with Examples and Templates)

    Write a research null hypothesis as a statement that the studied variables have no relationship to each other, or that there's no difference between 2 groups. Write a statistical null hypothesis as a mathematical equation, such as. μ 1 = μ 2 {\displaystyle \mu _ {1}=\mu _ {2}} if you're comparing group means.

  11. AP Biology Exam Tips

    Organize your answers as clearly and neatly as possible. You might want to label your answers according to the sub-part, such as (a), (b), (c), etc. This will assist you in organizing your thoughts, as well as helping to ensure that you answer all the parts of the free-response question. You should include the proper units for each number where ...

  12. How to Formulate a Null Hypothesis (With Examples)

    To distinguish it from other hypotheses, the null hypothesis is written as H 0 (which is read as "H-nought," "H-null," or "H-zero"). A significance test is used to determine the likelihood that the results supporting the null hypothesis are not due to chance. A confidence level of 95% or 99% is common. Keep in mind, even if the confidence level is high, there is still a small chance the ...

  13. AP Biology First Assessment Flashcards

    Study with Quizlet and memorize flashcards containing terms like Recap on How to write a null/alternate hypothesis, monomer of carbohydrates, dehydration reaction and more.

  14. PDF 2022 AP Exam Administration Student Samples and Commentary

    (c) State the null hypothesis for the experiment. • Temperature has no effect on the amount of light emitted. 1 point (d) A student claims that, as temperature increases, there will be an increase in the amount of light given off by the reaction in the first three seconds. Support the student's claim. Accept one of the following:

  15. Hypothesis Writing in AP Biology

    This video helps students to constuct their testable hypoteses and their working hypotheses.

  16. PDF AP Biology Student Samples from the 2023 Exam Administration

    In part (a) students were expected to describe that increased biodiversity results in increased ecosystem resilience (Skill 1.A; Learning Objective [LO] SYI-3.F from the AP Biology Course and Exam Description [CED]). In part (b) students were asked to justify the use of the control conditions in the experiment (Skill 3.C).

  17. How to Answer Experiment Questions on AP Biology FRQ

    2017 FRQ - #2 Bees and Caffeine Experiment. This question involves an experiment about bees and the nectar they encounter while pollinating flowers. The scientists want to understand the role of caffeine on the bees' memory. The question gives a table showing the results of the experiment, shown below.

  18. 4 Top Tips to Make AP Biology FRQs a Breeze

    AP Biology is known for being one of the tougher AP exams, and, for most students, the free-response section is the hardest part of the test.In 2021, the average score for every free-response question was less than a 50%! However, knowing what to expect can make it easier to get a great score on AP Biology FRQ. And in this guide, we explain everything you need to know to ace this section.

  19. Writing null and alternative hypotheses (practice)

    A ketchup company regularly receives large shipments of tomatoes. For each shipment that is received, a supervisor takes a random sample of 500 ‍ tomatoes to see what percent of the sample is bruised and performs a significance test. If the sample shows convincing evidence that more than 10 % ‍ of the entire shipment of tomatoes is bruised, then they will request a new shipment of tomatoes.

  20. What is a Null Hypothesis?

    Overview of null hypothesis, examples of null and alternate hypotheses, and how to write a null hypothesis statement.

  21. Null & Alternative Hypotheses

    The null and alternative hypotheses offer competing answers to your research question. When the research question asks "Does the independent variable affect the dependent variable?": The null hypothesis ( H0) answers "No, there's no effect in the population.". The alternative hypothesis ( Ha) answers "Yes, there is an effect in the ...

  22. How to establish a null hypothesis

    The statistician has determined that she will only reject the null hypothesis if she has 95% confidence that there is a relationship between variables. To have this level of confidence, the statistician must obtain a p value of 0.05 or lower. Therefore, she should not reject the null hypothesis since 0.1 is greater that 0.05.

  23. Examples of null and alternative hypotheses

    It is the opposite of your research hypothesis. The alternative hypothesis--that is, the research hypothesis--is the idea, phenomenon, observation that you want to prove. If you suspect that girls take longer to get ready for school than boys, then: Alternative: girls time > boys time. Null: girls time <= boys time.